You are on page 1of 45

PSAT/NMSQT

Questions and Answer Explanations 2010-2011

SATURDAY FORM

Writing Skills: Question 1


Difficulty Level: EASY (2 on a scale of 1-9)
Skill Category: Manage Grammatical Structures Used to Modify or Compare
Understand correct use of adjectives or adverbs, comparative structures (such as neither and nor), and
phrases used to modify or compare.
Question:

Select the option that, when used in place of the underlined material, produces the most effective sentence. If you think
the original sentence is best, select choice A.

Hearing the soldiers approaching was when Hua Mulan decided to save her elderly father by disguising herself as a man
and taking his place in the army.

(A) Hearing the soldiers approaching was when


(B) Hearing the soldiers approach, that was when
(C) She heard the soldiers approach, which was when
(D) When she heard the soldiers approaching,
(E) When she hears the soldiers approach,
Answer Explanation:

Corrected Sentence:
When she heard the soldiers approaching, Hua Mulan decided to save her elderly father by disguising herself as a man and
taking his place in the army.
Choice (D) is correct. It avoids the unidiomatic, illogical phrasing of the original by providing an appropriate modifying
clause (When she heard the soldiers approaching,) to indicate the moment in time at which Hua Mulan, the subject of
the sentence, decided to save her elderly father.
Choice (A) results in an unidiomatic, illogical construction. The subject of this version of the sentence, the gerund
Hearing, should not be used with the phrase was when It does not make sense to use the relative pronoun when
to refer to hearing. Hearing the soldiers approaching is not a moment in time.
Choice (B) results in a modification error and a pronoun error. It does not make sense to say that the pronoun that is
Hearing the soldiers approach. It is Hua Mulan who hears the soldiers approach. In addition, there is nothing in the
sentence to which that can logically refer.
Choice (C) results in a pronoun error. There is nothing in the sentence to which the pronoun which can logically refer.
Choice (E) results in a verb tense error. The present tense verb hears is not consistent with the past tense main verb
decided.

The College Board 2011. For the sole use of the person for whom this report has been provided. Not for duplication or distribution.

Page 1

PSAT/NMSQT

Questions and Answer Explanations 2010-2011

SATURDAY FORM

Writing Skills: Question 2


Difficulty Level: EASY (1 on a scale of 1-9)
Skill Category: Manage Word Choice and Grammatical Relationships Between Words
Understand relationships between and among words including subject-verb agreement, pronoun reference,
and verb form and tense.
Question:

Select the option that, when used in place of the underlined material, produces the most effective sentence. If you think
the original sentence is best, select choice A.

Recent air quality tests indicating that public buildings where indoor smoking is banned have indoor air pollution levels
up to 82 percent lower than those where indoor smoking is permitted.

(A) indicating that


(B) indicate that
(C) which indicate that
(D) indicating
(E) that indicate
Answer Explanation:

Corrected Sentence:
Recent air quality tests indicate that public buildings where indoor smoking is banned have indoor air pollution levels up
to 82 percent lower than those where indoor smoking is permitted.
Choice (B) is correct. It avoids the sentence fragment error of the original by replacing the present participle indicating
with the present tense main verb indicate.
Choice (A) results in a sentence fragment error. There is no main verb to carry out the action of the sentence, only the
present participle indicating. The present tense verb have cannot function as the main verb because Recent air
quality tests cannot have indoor air pollution levels.
Choice (C) results in a sentence fragment error. There is no main verb to carry out the action of the sentence; the phrase
which indicate that introduces a relative clause. The present tense verb have cannot function as the main verb because
Recent air quality tests cannot have indoor air pollution levels.
Choice (D) results in a sentence fragment error. There is no main verb to carry out the action of the sentence, only the
present participle indicating. The present tense verb have cannot function as the main verb because Recent air
quality tests cannot have indoor air pollution levels.
Choice (E) results in a sentence fragment error. There is no main verb to carry out the action of the sentence; the phrase
that indicate introduces a relative clause. The present tense verb have cannot function as the main verb because
Recent air quality tests cannot have indoor air pollution levels.

The College Board 2011. For the sole use of the person for whom this report has been provided. Not for duplication or distribution.

Page 2

PSAT/NMSQT

Questions and Answer Explanations 2010-2011

SATURDAY FORM

Writing Skills: Question 3


Difficulty Level: EASY (2 on a scale of 1-9)
Skill Category: Manage Phrases and Clauses in a sentence
Use well-formed sentence structures (e.g., parallelism, connectives, and relative clauses) to indicate
relationships between and among sentence elements.
Question:

Select the option that, when used in place of the underlined material, produces the most effective sentence. If you think
the original sentence is best, select choice A.

By them feeding on dead animals, the bald eagle plays an important role in natures cleanup process.

(A) By them feeding


(B) By their feeding
(C) By feeding
(D) They feed
(E) It feeds
Answer Explanation:

Corrected Sentence:
By feeding on dead animals, the bald eagle plays an important role in natures cleanup process.
Choice (C) is correct. It avoids the unidiomatic phrasing and pronoun agreement error of the original by placing the
gerund feeding directly after the preposition By to serve as its object.
Choice (A) results in unidiomatic phrasing and a pronoun agreement error. The gerund feeding serves as the object of
the preposition By and should directly follow it. In addition, the plural pronoun them does not agree with the singular
noun to which it refers, bald eagle.
Choice (B) results in unidiomatic phrasing and a pronoun agreement error. The gerund feeding serves as the object of
the preposition By and should directly follow it. In addition, the possessive plural pronoun their does not agree with
the singular noun to which it refers, bald eagle.
Choice (D) results in a comma splice and pronoun agreement error. Two independent clauses (They feed on dead
animals and The bald eagle plays an important role in natures cleanup process) are improperly joined by only a
comma. In addition, the plural pronoun They does not agree with the singular noun to which it refers, bald eagle.
Choice (E) results in a comma splice. Two independent clauses (It feeds on dead animals and The bald eagle plays an
important role in natures cleanup process) are improperly joined by only a comma.

The College Board 2011. For the sole use of the person for whom this report has been provided. Not for duplication or distribution.

Page 3

PSAT/NMSQT

Questions and Answer Explanations 2010-2011

SATURDAY FORM

Writing Skills: Question 4


Difficulty Level: EASY (3 on a scale of 1-9)
Skill Category: Recognize Correctly Formed Sentences
Recognize correct sentence structure.
Question:

Select the option that, when used in place of the underlined material, produces the most effective sentence. If you think
the original sentence is best, select choice A.

In the nineteenth century, scholars were amazed to discover that the Sumerian epic of Gilgamesh contains a story of a
great flood similar to the biblical story of Noah and the Flood .

(A) contains a story of a great flood similar to the biblical story of Noah and the Flood
(B) containing a flood story similar to the biblical story of Noah and the Flood
(C) and it contained a flood story similar to the biblical story of Noah and the Flood
(D) being similar to the biblical story of Noah and the Flood which also contained a flood story
(E) contains a story of a great flood, it was like the biblical story of Noah and the Flood
Answer Explanation:

Corrected Sentence:
In the nineteenth century, scholars were amazed to discover that the Sumerian epic of Gilgamesh contains a story of a
great flood similar to the biblical story of Noah and the Flood.
Choice (A) is correct. It avoids the errors in sentence structure of all the other options.
Choice (B) results in a faulty relative clause structure. The relative clause that begins that the Sumerian epic requires a
present tense verb (contains) to carry out the action of the subject epic, not the present participle containing.
Choice (C) results in a faulty relative clause structure. The relative clause that begins that the Sumerian epic requires a
present tense verb to carry out the action of the subject epic. To end the clause at Gilgamesh and begin a new clause
with and it contained leaves the relative clause incomplete.
Choice (D) results in a faulty relative clause structure. The relative clause that begins that the Sumerian epic requires a
present tense verb to carry out the action of the subject epic, not a present participle (being).
Choice (E) results in a comma splice error. Two independent clauses (In the nineteenth century, scholars were amazed to
discover that the Sumerian epic of Gilgamesh contains a story of a great flood and it was like the biblical story of Noah
and the Flood) are improperly joined by only a comma.

The College Board 2011. For the sole use of the person for whom this report has been provided. Not for duplication or distribution.

Page 4

PSAT/NMSQT

Questions and Answer Explanations 2010-2011

SATURDAY FORM

Writing Skills: Question 5


Difficulty Level: EASY (3 on a scale of 1-9)
Skill Category: Manage Phrases and Clauses in a sentence
Use well-formed sentence structures (e.g., parallelism, connectives, and relative clauses) to indicate
relationships between and among sentence elements.
Question:

Select the option that, when used in place of the underlined material, produces the most effective sentence. If you think
the original sentence is best, select choice A.

Bharati Mukherjees book The Middleman and Other Stories was widely admired and it being the first work by a
naturalized American citizen to win the National Book Critics Circle Award for Fiction.

(A) book The Middleman and Other Stories was widely admired and it being
(B) book, which was The Middleman and Other Stories , was widely admired and which was
(C) book The Middleman and Other Stories , widely admired and
(D) widely admired book The Middleman and Other Stories was
(E) widely admired book The Middleman and Other Stories , it was
Answer Explanation:

Corrected Sentence:
Bharati Mukherjees widely admired book The Middleman and Other Stories was the first work by a naturalized
American citizen to win the National Book Critics Circle Award for Fiction.
Choice (D) is correct. It avoids the faulty sentence structure of the original by constructing the sentence as one
independent clause with a main verb (Bharati Mukherjees widely admired bookwas), instead of using the
conjunction and to join an independent clause (Bharati Mukherjees bookwas widely admired) to a fragment (it
being the first workto win the National Book Critics Award for Fiction).
Choice (A) results in faulty sentence structure. An independent clause (Bharati Mukherjees bookwas widely
admired) and a fragment (it being the first workto win the National Book Critics Award for Fiction) are improperly
joined by the conjunction and.
Choice (B) results in faulty sentence structure. An independent clause (Bharati Mukherjees bookwas widely
admired) and a relative clause (which was the first workto win the National Book Critics Award for Fiction) are
improperly joined by the conjunction and.
Choice (C) results in a sentence fragment error. There is no main verb to carry out the action of the sentence, only the past
participle admired.
Choice (E) results in a wordiness error. The pronoun it, which refers to the subject Bharati Mukherjees widely
admired book The Middleman and Other Stories , is unnecessary and should be deleted, along with the comma
preceding it.

The College Board 2011. For the sole use of the person for whom this report has been provided. Not for duplication or distribution.

Page 5

PSAT/NMSQT

Questions and Answer Explanations 2010-2011

SATURDAY FORM

Writing Skills: Question 6


Difficulty Level: EASY (3 on a scale of 1-9)
Skill Category: Recognize Correctly Formed Sentences
Recognize correct sentence structure.
Question:

Select the option that, when used in place of the underlined material, produces the most effective sentence. If you think
the original sentence is best, select choice A.

When Ada Byron learned of Charles Babbages Analytical Engine, a precursor of modern computers, she correctly
predicted that machines like it would one day produce music.

(A) learned of
(B) learns of
(C) has learned of
(D) had learned that
(E) learning that
Answer Explanation:

Corrected Sentence:
When Ada Byron learned of Charles Babbages Analytical Engine, a precursor of modern computers, she correctly
predicted that machines like it would one day produce music.
Choice (A) is correct. It avoids the errors of all the other options by using the past tense verb learned, which is
consistent with the past tense verb predicted, and the idiomatic preposition of, which appropriately joins learned to
the noun phrase Charles Babbages Analytical Engine.
Choice (B) results in a verb tense error. The present tense verb learns is not consistent with the past tense verb
predicted and should be replaced with the past tense verb learned.
Choice (C) results in a verb tense error. The present perfect tense verb has learned is not consistent with the past tense
verb predicted and should be replaced with the past tense verb learned.
Choice D results in a verb tense error and unidiomatic phrasing. The past perfect tense verb had learned is not
appropriate in this context, because Ada Byrons learning about Charles Babbages Analytical Engine happened at or
around the same time as her predict[ing] that machines like it would one day produce music, as indicated by the relative
pronoun When. The past tense verb learned is required to be consistent with the past tense verb predicted. In
addition, the word that after learned normally indicates the beginning of a relative clause, but there is no relative
clause in the first part of the sentence. Instead, the preposition of is required.
Choice (E) results in a verb tense error and unidiomatic phrasing. The present participle learning is not consistent with
the past tense verb predicted and should be replaced with the past tense verb learned. In addition, the word that
after learned normally indicates the beginning of a relative clause, but there is no relative clause in the first part of the
sentence. Instead, the preposition of is required.

The College Board 2011. For the sole use of the person for whom this report has been provided. Not for duplication or distribution.

Page 6

PSAT/NMSQT

Questions and Answer Explanations 2010-2011

SATURDAY FORM

Writing Skills: Question 7


Difficulty Level: MEDIUM (4 on a scale of 1-9)
Skill Category: Manage Grammatical Structures Used to Modify or Compare
Understand correct use of adjectives or adverbs, comparative structures (such as neither and nor), and
phrases used to modify or compare.
Question:

Select the option that, when used in place of the underlined material, produces the most effective sentence. If you think
the original sentence is best, select choice A.

Labor leader Cesar Chavez began his career as an activist by helping many California farmworkers to register to vote,
which was successful .

(A) activist by helping many California farmworkers to register to vote, which was successful
(B) activist by helping many California farmworkers registering to vote, which he was successful
(C) activist by successfully helping many California farmworkers register to vote
(D) activist, he successfully helped many California farmworkers register to vote
(E) activist with successfully helping many California farmworkers register to be a voter
Answer Explanation:

Corrected Sentence:
Labor leader Cesar Chavez began his career as an activist by successfully helping many California farmworkers register to
vote.
Choice (C) is correct. It avoids the modification error of the original by replacing the relative clause which was
successful with the adverb successfully and moving it before the present participle it modifies, helping.
Choice (A) results in a modification error. The relative clause which was successful, when placed at the end of the
sentence, fails to indicate what, exactly, was successful. The modifier should be replaced with the adverb successfully
and placed immediately before the participle helping.
Choice (B) results in unidiomatic phrasing. The verb preceding the infinitive to vote should be the present tense verb
register, not the present participle registering. In addition, the relative clause which he was successful is
unidiomatic on its own and should be completed with a phrase such as in doing.
Choice (D) results in a comma splice. Two independent clauses (Labor leader Cesar Chavez began his career as an
activist and he successfully helped many California farmworkers register to vote) are improperly joined by only a
comma.
Choice (E) results in an unidiomatic preposition and a noun agreement error. The preposition with is not idiomatic when
joining the phrase began his career to the phrase successfully helping and should be replaced with the preposition
by. In addition, the singular noun a voter is not consistent with the plural noun farmworkers and should be replaced
with voters (of course, a more idiomatic way to say register to be voters would be register to vote).

The College Board 2011. For the sole use of the person for whom this report has been provided. Not for duplication or distribution.

Page 7

PSAT/NMSQT

Questions and Answer Explanations 2010-2011

SATURDAY FORM

Writing Skills: Question 8


Difficulty Level: MEDIUM (5 on a scale of 1-9)
Skill Category: Manage Grammatical Structures Used to Modify or Compare
Understand correct use of adjectives or adverbs, comparative structures (such as neither and nor), and
phrases used to modify or compare.
Question:

Select the option that, when used in place of the underlined material, produces the most effective sentence. If you think
the original sentence is best, select choice A.

Finding its way through sediment on the sea floor, the 400700 whiskers on a walrus snout are used to sense the
presence of clams and other invertebrates on which it feeds.

(A) the 400700 whiskers on a walrus snout are used


(B) the walrus uses the 400700 whiskers on its snout
(C) walruses use the 400700 whiskers on their snout
(D) a walrus snout, having 400700 whiskers, is used
(E) the snouts of walruses use 400700 whiskers
Answer Explanation:

Corrected Sentence:
Finding its way through sediment on the sea floor, the walrus uses the 400-700 whiskers on its snout to sense the presence
of clams and other invertebrates on which it feeds.
Choice (B) is correct. It avoids the modification error of the original by placing the correct subject the walrus
immediately after the modifying phrase Finding its way through sediment on the sea floor.
Choice (A) results in a modification error. It does not make sense to say that the 400-700 whiskers are Finding its way
through sediment on the sea floor. In addition, the singular possessive pronoun its does not agree with the plural
subject whiskers.
Choice (C) results in a pronoun agreement error. The plural noun walruses and the plural pronoun their do not agree
with the singular pronouns its and it.
Choice (D) results in a modification error. It is not a walrus snout that is Finding its way through sediment on the sea
floor and feeds on clams and other invertebrates, but the walrus itself, which uses its snout to sense the presence of
these food sources.
Choice (E) results in a modification error and pronoun agreement error. It is not the snouts that are Finding [their] way
through sediment on the sea floor, but the walrus itself, which uses its snout to sense the presence of clams In
addition, the plural noun snouts does not agree with the singular pronouns its and it.

The College Board 2011. For the sole use of the person for whom this report has been provided. Not for duplication or distribution.

Page 8

PSAT/NMSQT

Questions and Answer Explanations 2010-2011

SATURDAY FORM

Writing Skills: Question 9


Difficulty Level: MEDIUM (5 on a scale of 1-9)
Skill Category: Manage Phrases and Clauses in a sentence
Use well-formed sentence structures (e.g., parallelism, connectives, and relative clauses) to indicate
relationships between and among sentence elements.
Question:

Select the option that, when used in place of the underlined material, produces the most effective sentence. If you think
the original sentence is best, select choice A.

Although Jonas Salk received great acclaim for developing a vaccine that virtually eliminated polio in the United States,
but his breakthrough was made possible by the work of earlier scientists.

(A) States, but his breakthrough was made possible


(B) States; his breakthrough was made possible, however,
(C) States; yet his breakthrough was made possible
(D) States, his breakthrough was made possible
(E) States, making his breakthrough possible was
Answer Explanation:

Corrected Sentence:
Although Jonas Salk received great acclaim for developing a vaccine that virtually eliminated polio in the United States,
his breakthrough was made possible by the work of earlier scientists.
Choice (D) is correct. It avoids the coordination error of the original by deleting the unnecessary additional conjunction
but.
Choice (A) results in a coordination error. The conjunction but is not necessary in the sentence and should be deleted.
The conjunction Although already indicates the contrast between the two clauses (Jonas Salk received great acclaim
for developing a vaccine that virtually eliminated polio in the United States and his breakthrough was made possible by
the work of earlier scientists).
Choice (B) results in a coordination error and improper use of a semicolon. The conjunction however is not necessary
and should be deleted, since the conjunction Although already indicates the contrast between the two clauses (Jonas
Salk received great acclaim for developing a vaccine that virtually eliminated polio and his breakthrough was made
possible by the work of earlier scientists). In addition, the dependent clause that begins Although Jonas Salk received in
the United States should be followed by a comma, not a semicolon. A semicolon should be used to join two independent
clauses.
Choice (C) results in a coordination error and improper use of a semicolon. The conjunction yet is not necessary and
should be deleted, since the conjunction Although already indicates the contrast between the two clauses (Jonas Salk
received great acclaim for developing a vaccine that virtually eliminated polio in the United States and his breakthrough
was made possible by the work of earlier scientists). In addition, the dependent clause that begins Although Jonas Salk
received should be followed by a comma, not a semicolon. A semicolon should be used to join two independent
clauses.
Choice (E) results in unidiomatic phrasing. In the clause making his breakthrough possible was by the work of earlier
scientists, the construction was by is unidiomatic. The phrase would be idiomatic, though still awkward, if the
preposition by were deleted. A better, more straightforward phrasing would be his breakthrough was made possible by
The College Board 2011. For the sole use of the person for whom this report has been provided. Not for duplication or distribution.

Page 9

PSAT/NMSQT

Questions and Answer Explanations 2010-2011

SATURDAY FORM

the work of earlier scientists.

The College Board 2011. For the sole use of the person for whom this report has been provided. Not for duplication or distribution.

Page 10

PSAT/NMSQT

Questions and Answer Explanations 2010-2011

SATURDAY FORM

Writing Skills: Question 10


Difficulty Level: MEDIUM (5 on a scale of 1-9)
Skill Category: Manage Word Choice and Grammatical Relationships Between Words
Understand relationships between and among words including subject-verb agreement, pronoun reference,
and verb form and tense.
Question:

Select the option that, when used in place of the underlined material, produces the most effective sentence. If you think
the original sentence is best, select choice A.

Able to swim by rapidly opening and closing their shell , a scallop can flee predators more effectively than can a clam or
mussel.

(A) rapidly opening and closing their shell


(B) rapidly opening and closing their shells
(C) rapidly opening and closing its shell
(D) rapid openings and closings of their shells
(E) the rapidly opening and closing of its shell
Answer Explanation:

Corrected Sentence:
Able to swim by rapidly opening and closing its shell, a scallop can flee predators more effectively than can a clam or
mussel.
Choice (C) is correct. It avoids the pronoun agreement error of the original by providing the singular pronoun its to
agree with the singular subject a scallop.
Choice (A) results in a pronoun agreement error. The plural pronoun their does not agree with the singular subject a
scallop and should be replaced with the singular pronoun its.
Choice (B) results in a pronoun agreement error. The plural pronoun their does not agree with the singular subject a
scallop and should be replaced with the singular pronoun its.
Choice (D) results in unidiomatic phrasing and a pronoun agreement error. The plural gerunds openings and closings
are not idiomatic after the preposition by; instead, the singular forms (opening and closing), which indicate ongoing
actions, should be used. In addition, the plural pronoun their does not agree with the singular subject a scallop and
should be replaced with the singular pronoun its.
Choice (E) results in the unidiomatic use of an adverb. In this version of the sentence, the adverb rapidly modifies the
gerunds opening and closing. Although an adverb can be used to modify a gerund, the addition of the article the
and the preposition of make the use of rapidly unidiomatic in this case. The adjective rapid should be used instead.

The College Board 2011. For the sole use of the person for whom this report has been provided. Not for duplication or distribution.

Page 11

PSAT/NMSQT

Questions and Answer Explanations 2010-2011

SATURDAY FORM

Writing Skills: Question 11


Difficulty Level: MEDIUM (6 on a scale of 1-9)
Skill Category: Manage Grammatical Structures Used to Modify or Compare
Understand correct use of adjectives or adverbs, comparative structures (such as neither and nor), and
phrases used to modify or compare.
Question:

Select the option that, when used in place of the underlined material, produces the most effective sentence. If you think
the original sentence is best, select choice A.

A best seller when it was first published more than 45 years ago, many still regard Rachel Carsons Silent Spring as the
cornerstone of the environmentalist movement.

(A) many still regard Rachel Carsons Silent Spring as


(B) many still regard Rachel Carsons Silent Spring to be
(C) there are many who still regard Rachel Carsons Silent Spring as
(D) Rachel Carsons Silent Spring is still regarded by many as
(E) Rachel Carsons Silent Spring , which is still regarded by many to be
Answer Explanation:

Corrected Sentence:
A best seller when it was first published more than 45 years ago, Rachel Carsons Silent Spring is still regarded by many
as the cornerstone of the environmentalist movement.
Choice (D) is correct. It avoids the modification error of the original by placing the subject of the sentence, Rachel
Carsons Silent Spring , immediately after the modifying phrase A best seller when it was first published more than 45
years ago.
Choice (A) results in a modification error. It does not make sense to say that many is A best seller when it was first
published more than 45 years ago.
Choice (B) results in a modification error. It does not make sense to say that many is A best seller when it was first
published more than 45 years ago.
Choice (C) results in a modification error. It does not make sense to say that there is A best seller when it was first
published more than 45 years ago.
Choice (E) results in a sentence fragment. There is no main verb to carry out the action of the sentence, only the relative
clause that begins which is still regarded

The College Board 2011. For the sole use of the person for whom this report has been provided. Not for duplication or distribution.

Page 12

PSAT/NMSQT

Questions and Answer Explanations 2010-2011

SATURDAY FORM

Writing Skills: Question 12


Difficulty Level: MEDIUM (5 on a scale of 1-9)
Skill Category: Recognize Correctly Formed Sentences
Recognize correct sentence structure.
Question:

Select the option that, when used in place of the underlined material, produces the most effective sentence. If you think
the original sentence is best, select choice A.

Most of the founders of the United States had hoped that the new country could be governed without political parties.

(A) could be
(B) could be able to be
(C) could have been able to be
(D) can be
(E) will be
Answer Explanation:

Corrected Sentence:
Most of the founders of the United States had hoped that the new country could be governed without political parties.
Choice (A) is correct. It avoids the verb tense errors of all the other options by providing the past tense verb phrase could
be, which is consistent with the past perfect verb phrase had hoped.
Choice (B) results in wordiness. The phrase able to be is unnecessary and should be deleted. The past tense verb phrase
could be is consistent with the past perfect verb phrase had hoped.
Choice (C) results in unidiomatic phrasing. The past tense verb phrase could have been able to be is not idiomatic and
should be replaced with the past tense verb phrase could be in order to be consistent with the past perfect verb phrase
had hoped.
Choice (D) results in a verb tense error. The present tense verb can be is not consistent with the past perfect verb phrase
had hoped and should be replaced with the past tense verb phrase could be.
Choice (E) results in a verb tense error. The future tense verb will be is not consistent with the past perfect verb phrase
had hoped and should be replaced with the past tense verb phrase could be.

The College Board 2011. For the sole use of the person for whom this report has been provided. Not for duplication or distribution.

Page 13

PSAT/NMSQT

Questions and Answer Explanations 2010-2011

SATURDAY FORM

Writing Skills: Question 13


Difficulty Level: MEDIUM (6 on a scale of 1-9)
Skill Category: Manage Word Choice and Grammatical Relationships Between Words
Understand relationships between and among words including subject-verb agreement, pronoun reference,
and verb form and tense.
Question:

Select the option that, when used in place of the underlined material, produces the most effective sentence. If you think
the original sentence is best, select choice A.

Until about 135 years ago, when the remains of an ancient city were discovered on the plain of Troy, there has been a
general assumption that the events depicted in Homers Iliad are purely fictitious.

(A) there has been a general assumption that the events depicted in Homers Iliad are
(B) it had generally been assumed that the events depicted in Homers Iliad were
(C) the general assumption was for the events depicted in Homers Iliad to have been
(D) the events depicted in Homers Iliad have generally been assumed to be
(E) Homers Iliad is generally assumed to have depicted events that are
Answer Explanation:

Corrected Sentence:
Until about 135 years ago, when the remains of an ancient city were discovered on the plain of Troy, it had generally been
assumed that the events depicted in Homers Iliad were purely fictitious.
Choice (B) is correct. It avoids the verb tense error of the original by providing the past perfect verb phrase hadbeen
assumed and the past tense verb were to be consistent with the past tense verb phrase were discovered (when the
remains of an ancient city were discovered).
Choice (A) results in verb tense errors. The present tense verbs has been and are are not consistent with the past tense
verb phrase were discovered (when the remains of an ancient city were discovered). They should be replaced with
the verbs had been and were, respectively.
Choice (C) results in unidiomatic phrasing and a verb tense error. The preposition for is not idiomatic in this context
and should be replaced with the relative pronoun that. In addition, the present perfect infinitive to have been is not
appropriate because a relative clause requires a main verb; therefore, to have been should be replaced with the past
tense verb were.
Choice (D) results in a verb tense error. The present perfect verb phrase havebeen assumed is inconsistent with the
past tense verb phrase were discovered (when the remains of an ancient city were discovered). It should be
replaced with the past perfect verb phrase hadbeen assumed.
Choice (E) results in verb tense errors. The present tense verbs isassumed and are are not consistent with the past
tense verb phrase were discovered (when the remains of an ancient city were discovered). They should be replaced
with the past tense verbs wasassumed and were, respectively.

The College Board 2011. For the sole use of the person for whom this report has been provided. Not for duplication or distribution.

Page 14

PSAT/NMSQT

Questions and Answer Explanations 2010-2011

SATURDAY FORM

Writing Skills: Question 14


Difficulty Level: MEDIUM (6 on a scale of 1-9)
Skill Category: Manage Phrases and Clauses in a sentence
Use well-formed sentence structures (e.g., parallelism, connectives, and relative clauses) to indicate
relationships between and among sentence elements.
Question:

Select the option that, when used in place of the underlined material, produces the most effective sentence. If you think
the original sentence is best, select choice A.

Even his closest friends believe that the candidate is unethical, they acknowledge that this lack of integrity may cause
him to lose the election.

(A) they acknowledge that this lack of integrity


(B) they acknowledge that to lack integrity
(C) which they acknowledge that this lack of integrity
(D) and they acknowledge that this lack of integrity
(E) acknowledging this lacking of integrity
Answer Explanation:

Corrected Sentence:
Even his closest friends believe that the candidate is unethical, and they acknowledge that his lack of integrity may cause
him to lose the election.
Choice (D) is correct. It avoids the comma splice error of the original by providing the conjunction and to join the two
independent clauses (Even his closest friends believe that the candidate is unethical and they acknowledge that this
lack of integrity may cause him to lose the election).
Choice (A) results in a comma splice error. Two independent clauses (Even his closest friends believe that the candidate
is unethical and they acknowledge that this lack of integrity may cause him to lose the election) are improperly joined
by only a comma.
Choice (B) results in a comma splice error and unidiomatic phrasing. Two independent clauses (Even his closest friends
believe that the candidate is unethical and they acknowledge that to lack integrity may cause him to lose the election)
are improperly joined by only a comma. In addition, the infinitive phrase to lack integrity is unidiomatic in the context
of the second clause and should be replaced with this lack of.
Choice (C) results in wordiness and an unclear sentence. The phrase that this lack of integrity is unnecessary and should
be deleted. In addition, it is unclear what is being acknowledged in the second part of the sentence.
Choice (E) results in a comma splice error. Two independent clauses (Even his closest friends believe that the candidate
is unethical and acknowledging this lacking of integrity may cause him to lose the election) are improperly joined by
only a comma.

The College Board 2011. For the sole use of the person for whom this report has been provided. Not for duplication or distribution.

Page 15

PSAT/NMSQT

Questions and Answer Explanations 2010-2011

SATURDAY FORM

Writing Skills: Question 15


Difficulty Level: HARD (7 on a scale of 1-9)
Skill Category: Manage Phrases and Clauses in a sentence
Use well-formed sentence structures (e.g., parallelism, connectives, and relative clauses) to indicate
relationships between and among sentence elements.
Question:

Select the option that, when used in place of the underlined material, produces the most effective sentence. If you think
the original sentence is best, select choice A.

Since hot wax does not adhere to it, that is why the mineral carnelian was used in ancient times to make signet rings for
imprinting wax seals on letters.

(A) Since hot wax does not adhere to it, that is why
(B) Being known that hot wax does not adhere to it,
(C) Knowing that hot wax does not adhere to it,
(D) Because hot wax does not adhere to it,
(E) Hot wax does not adhere to it, therefore
Answer Explanation:

Corrected Sentence:
Because hot wax does not adhere to it, the mineral carnelian was used in ancient times to make signet rings for imprinting
wax seals on letters.
Choice (D) is correct. It avoids the wordiness of the original by removing the unnecessary phrase that is why. Either
since or because would be correct in this context, but because makes the cause-effect relationship unambiguously
clear.
Choice (A) results in wordiness. The phrase that is why is unnecessary and should be deleted.
Choice (B) results in unidiomatic phrasing. The phrase Being known that is unidiomatic and should be replaced with
since or because.
Choice (C) results in a modification error. It does not make sense to say that the mineral carnelian can [know] that hot
wax does not adhere to it.
Choice (E) results in the improper use of commas. In this sentence, the conjunction therefore requires a semicolon
before it and a comma after it.

The College Board 2011. For the sole use of the person for whom this report has been provided. Not for duplication or distribution.

Page 16

PSAT/NMSQT

Questions and Answer Explanations 2010-2011

SATURDAY FORM

Writing Skills: Question 16


Difficulty Level: MEDIUM (6 on a scale of 1-9)
Skill Category: Manage Word Choice and Grammatical Relationships Between Words
Understand relationships between and among words including subject-verb agreement, pronoun reference,
and verb form and tense.
Question:

Select the option that, when used in place of the underlined material, produces the most effective sentence. If you think
the original sentence is best, select choice A.

If she would have been offered an internship in Atlanta, Tamika would have to rent out her apartment in Chicago.

(A) If she would have been offered


(B) Were she to be offered
(C) With it being offered her
(D) Offering her
(E) By being offered
Answer Explanation:

Corrected Sentence:
Were she to be offered an internship in Atlanta, Tamika would have to rent out her apartment in Chicago.
Choice (B) is correct. It avoids the verb tense error of the original by providing the subjunctive verb in an appropriate
alternative to the if-clause: Were she to be offered
Choice (A) results in a verb tense error. The conditional verb phrase would have in the main clause (Tamika would
have to rent out her apartment in Chicago) requires the subjunctive verb tense in its if-clause rather than the
conditional verb tense. Thus, If she would have been offered (conditional) should be changed to If she were to be
offered (subjunctive) or the alternative inverted if-clause, Were she to be offered (subjunctive).
Choice (C) results in unidiomatic phrasing. The phrase With it being offered her is unidiomatic and should be replaced
with an if-clause that uses the subjunctive verb tense: If she were to be offered or Were she to be offered.
Choice (D) results in a modification error. It does not make sense to say that Tamika is Offering [herself] an internship in
Atlanta.
Choice (E) results in unidiomatic phrasing. The phrase By being offered is unidiomatic and should be replaced with an
if-clause that uses the subjunctive verb tense: If she were to be offered or Were she to be offered.

The College Board 2011. For the sole use of the person for whom this report has been provided. Not for duplication or distribution.

Page 17

PSAT/NMSQT

Questions and Answer Explanations 2010-2011

SATURDAY FORM

Writing Skills: Question 17


Difficulty Level: HARD (7 on a scale of 1-9)
Skill Category: Manage Grammatical Structures Used to Modify or Compare
Understand correct use of adjectives or adverbs, comparative structures (such as neither and nor), and
phrases used to modify or compare.
Question:

Select the option that, when used in place of the underlined material, produces the most effective sentence. If you think
the original sentence is best, select choice A.

Environmentalists have claimed that someone who drives an SUV instead of a fuel-efficient vehicle wastes more energy
in a single year than leaving a refrigerator door open for six years.

(A) someone who drives an SUV instead of a fuel-efficient vehicle wastes more energy in a single year than
leaving
(B) someone driving an SUV instead of a fuel-efficient vehicle wastes more energy in a single year than if
they leave
(C) to drive an SUV instead of a fuel-efficient vehicle for a single year wastes more energy than by leaving
(D) by driving an SUV instead of a fuel-efficient vehicle more energy is wasted in a single year than leaving
(E) driving an SUV instead of a fuel-efficient vehicle for a single year wastes more energy than leaving
Answer Explanation:

Corrected Sentence:
Environmentalists have claimed that driving an SUV instead of a fuel-efficient vehicle for a single year wastes more
energy than leaving a refrigerator door open for six years.
Choice (E) is correct. It avoids the illogical comparison and lack of parallelism of the original by comparing driving an
SUV to leaving a refrigerator door open, instead of someone who drives an SUV to leaving a refrigerator door
open. In addition, when two things are being compared in a sentence, they should be grammatically parallel.
Choice (A) results in an illogical comparison and a lack of parallelism. It does not make sense to compare someone who
drives an SUV to leaving a refrigerator door open. In addition, when two things are being compared in a sentence, they
should be grammatically parallel.
Choice (B) results in a pronoun agreement error and lack of parallelism. The plural pronoun they does not agree with
the singular noun someone. In addition, the gerund phrase someone driving is not parallel with the if-clause if
they leave When two things are being compared in a sentence, they should be grammatically parallel.
Choice (C) results in a lack of parallelism. The infinitive phrase to drive an SUV is not parallel with the prepositional
phrase by leaving a refrigerator door open. When two things are being compared in a sentence, they should be
grammatically parallel.
Choice (D) results in a lack of parallelism. The prepositional phrase by driving an SUV is not parallel with the gerund
phrase leaving a refrigerator door open. When two things are being compared in a sentence, they should be
grammatically parallel.

The College Board 2011. For the sole use of the person for whom this report has been provided. Not for duplication or distribution.

Page 18

PSAT/NMSQT

Questions and Answer Explanations 2010-2011

SATURDAY FORM

Writing Skills: Question 18


Difficulty Level: HARD (7 on a scale of 1-9)
Skill Category: Manage Phrases and Clauses in a sentence
Use well-formed sentence structures (e.g., parallelism, connectives, and relative clauses) to indicate
relationships between and among sentence elements.
Question:

Select the option that, when used in place of the underlined material, produces the most effective sentence. If you think
the original sentence is best, select choice A.

Plays performed in fourteenth-century France featured a variety of songs, ranging from popular tunes sung by individual
actors in addition to religious hymns sung by groups.

(A) in addition to
(B) as well as
(C) and even
(D) and
(E) to
Answer Explanation:

Corrected Sentence:
Plays performed in fourteenth-century France featured a variety of songs, ranging from popular tunes sung by individual
actors to religious hymns sung by groups.
Choice (E) is correct. It avoids the correlative construction error of the original by replacing the phrase in addition to
with the preposition to, which appropriately completes the correlative construction ranging fromto.
Choice (A) results in a correlative construction error. The phrase in addition to is not the correct phrase to complete the
correlative construction ranging fromto and should be replaced with the preposition to.
Choice (B) results in a correlative construction error. The phrase as well as is not the correct phrase to complete the
correlative construction ranging fromto and should be replaced with the preposition to.
Choice (C) results in a correlative construction error. The phrase and even is not the correct phrase to complete the
correlative construction ranging fromto and should be replaced with the preposition to.
Choice (D) results in a correlative construction error. The conjunction and is not the correct part of speech to complete
the correlative construction ranging fromto and should be replaced with the preposition to.

The College Board 2011. For the sole use of the person for whom this report has been provided. Not for duplication or distribution.

Page 19

PSAT/NMSQT

Questions and Answer Explanations 2010-2011

SATURDAY FORM

Writing Skills: Question 19


Difficulty Level: HARD (9 on a scale of 1-9)
Skill Category: Manage Grammatical Structures Used to Modify or Compare
Understand correct use of adjectives or adverbs, comparative structures (such as neither and nor), and
phrases used to modify or compare.
Question:

Select the option that, when used in place of the underlined material, produces the most effective sentence. If you think
the original sentence is best, select choice A.

The strange, nonsensical songs of the Icelandic rock group Sigur Rs made such an impression on Thom Yorke so that he
invited the band to join Radioheads European concert tour.

(A) made such an impression on Thom Yorke so that he invited


(B) was impressive enough for Thom Yorke to invite
(C) being so impressive to Thom Yorke, who invited
(D) impressed Thom Yorke to where he invited
(E) so impressed Thom Yorke that he invited
Answer Explanation:

Corrected Sentence:
The strange, nonsensical songs of the Icelandic rock group Sigur Rs so impressed Thom Yorke that he invited the band
to join Radioheads European concert tour.
Choice (E) is correct. It avoids the correlative construction error of the original by changing the incorrect phrase made
such an impressionso that, which does not need the conjunction so, to the correct phrase so impressedthat,
which is also more concise than the original.
Choice (A) results in a correlative construction error. The correlative construction [verb] such that does not need
the conjunction so.
Choice (B) results in a subject-verb agreement error. The singular verb was does not agree with the subject of the
sentence, the plural noun songs.
Choice (C) results in a sentence fragment error. There is no main verb to carry out the action of the sentence, only the
present participle being.
Choice (D) results in unidiomatic phrasing. It is unidiomatic to say that the songs impressed Thom Yorke to where he
invited There is no place or location to which the relative pronoun where could logically refer. The phrase should be
replaced with were so impressive that Thom Yorke invited or so impressed Thom Yorke that he invited.

The College Board 2011. For the sole use of the person for whom this report has been provided. Not for duplication or distribution.

Page 20

PSAT/NMSQT

Questions and Answer Explanations 2010-2011

SATURDAY FORM

Writing Skills: Question 20


Difficulty Level: HARD (9 on a scale of 1-9)
Skill Category: Recognize Correctly Formed Sentences
Recognize correct sentence structure.
Question:

Select the option that, when used in place of the underlined material, produces the most effective sentence. If you think
the original sentence is best, select choice A.

The Romans occupied Britain in the first century A.D. but, perhaps because of the increasing ferocity of Germanic
invasions, ultimately abandoned the island in 410 .

(A) A.D. but, perhaps because of the increasing ferocity of Germanic invasions, ultimately abandoned the
island in 410
(B) A.D., the island was ultimately abandoned in 410, perhaps because of the increasing ferocity of Germanic
invasions
(C) A.D., ultimately perhaps because of the increasing ferocity of Germanic invasions, the island was
abandoned in 410
(D) A.D., but ultimately abandoning the island in 410 perhaps because of the increasing ferocity of Germanic
invasions
(E) A.D., however, the island was ultimately abandoned in 410, perhaps because of the increasing ferocity of
Germanic invasions
Answer Explanation:

Corrected Sentence:
The Romans occupied Britain in the first century A.D. but, perhaps because of the increasing ferocity of Germanic
invasions, ultimately abandoned the island in 410.
Choice (A) is correct. It avoids all the errors of the other options and appropriately coordinates the three clauses of the
sentence (The Romans occupied Britain in the first century A.D., perhaps because of the increasing ferocity of
Germanic invasions, and ultimately abandoned the island in 410).
Choice (B) results in a comma splice error. Two independent clauses (The Romans occupied Britain in the first century
A.D. and the island was ultimately abandoned in 410, perhaps because of the increasing ferocity of Germanic
invasions) are improperly joined by only a comma.
Choice (C) results in a comma splice error. Two independent clauses (The Romans occupied Britain in the first century
A.D. and ultimately perhaps because of the increasing ferocity of Germanic invasions, the island was abandoned in
410) are improperly joined by only a comma.
Choice (D) results in faulty sentence structure. There is no main verb following the conjunction but to carry out the
action of what should be the second independent clause, only the participle abandoning.
Choice (E) results in the improper use of the conjunction however. However requires a semicolon before it as well as
a comma after it in order to properly join two independent clauses.

The College Board 2011. For the sole use of the person for whom this report has been provided. Not for duplication or distribution.

Page 21

PSAT/NMSQT

Questions and Answer Explanations 2010-2011

SATURDAY FORM

Writing Skills: Question 21


Difficulty Level: EASY (1 on a scale of 1-9)
Skill Category: Manage Word Choice and Grammatical Relationships Between Words
Understand relationships between and among words including subject-verb agreement, pronoun reference,
and verb form and tense.
Question:

Select the one underlined part of the sentence that must be changed to make the sentence correct, or select "No error" to
indicate that the sentence contains no error.

Unwilling to delay the verdict any longer, (a) the other jurors and (b) me , deadlocked
proceedings, now (d) sought to resolve our disagreements. (e) No error

(c) throughout the

Correct Answer: B
Answer Explanation:

Corrected Sentence:
Unwilling to delay the verdict any longer, the other jurors and I, deadlocked throughout the proceedings, now sought to
resolve our disagreements.
The error in the sentence occurs at (B). The subject of the sentence is the other jurors and me. A pronoun that serves as
the subject of a sentence must be in the subjective case (also known as the nominative case), so me must be replaced
with I. Me is in the objective case, so it must function as an object of a verb or a preposition: me cannot be the
subject of a sentence.
There is no error at (A). The article the and the adjective other appropriately modify the noun jurors. The noun
phrase the other jurors helps to clarify that the pronoun me (which should be replaced with I) refers to a juror.
There is no error at (C). The preposition throughout appropriately introduces the prepositional phrase throughout the
proceedings, which appropriately indicates the duration of the jurors deadlock.
There is no error at (D). The past tense verb sought is consistent with the preceding past participle deadlocked, and
the infinitive form of the verb to resolve appropriately indicates what the jurors sought to do.
There is an error in the sentence.

The College Board 2011. For the sole use of the person for whom this report has been provided. Not for duplication or distribution.

Page 22

PSAT/NMSQT

Questions and Answer Explanations 2010-2011

SATURDAY FORM

Writing Skills: Question 22


Difficulty Level: MEDIUM (4 on a scale of 1-9)
Skill Category: Manage Word Choice and Grammatical Relationships Between Words
Understand relationships between and among words including subject-verb agreement, pronoun reference,
and verb form and tense.
Question:

Select the one underlined part of the sentence that must be changed to make the sentence correct, or select "No error" to
indicate that the sentence contains no error.

Katherine Cheung, who (a) had flew as a licensed pilot (b) throughout the 1930s, (c) was recognized as a pioneer
in aviation and (d) inducted into the Women in Aviation International Pioneer Hall of Fame in 2000. (e) No error

Correct Answer: A
Answer Explanation:

Corrected Sentence:
Katherine Cheung, who had flown as a licensed pilot throughout the 1930s, was recognized as a pioneer in aviation and
inducted into the Women in Aviation International Pioneer Hall of Fame in 2000.
The error in the sentence occurs at (A). The past perfect verb phrase had flown is required to indicate an action that took
place before another action in the past. Had flew is ungrammatical and must be replaced with had flown.
There is no error at (B). The preposition throughout appropriately introduces the prepositional phrase throughout the
1930s, which indicates the duration of Cheungs career as a pilot.
There is no error at (C). The past tense verb phrase was recognized is consistent with the past tense verb [was]
inducted that follows it in the sentence.
There is no error at (D). The past tense verb inducted is part of the appropriate past tense verb phrase was
recognizedand inducted, which indicates events that occurred in the past.
There is an error in the sentence.

The College Board 2011. For the sole use of the person for whom this report has been provided. Not for duplication or distribution.

Page 23

PSAT/NMSQT

Questions and Answer Explanations 2010-2011

SATURDAY FORM

Writing Skills: Question 23


Difficulty Level: EASY (3 on a scale of 1-9)
Skill Category: Manage Grammatical Structures Used to Modify or Compare
Understand correct use of adjectives or adverbs, comparative structures (such as neither and nor), and
phrases used to modify or compare.
Question:

Select the one underlined part of the sentence that must be changed to make the sentence correct, or select "No error" to
indicate that the sentence contains no error.

(a) Current opposition to the publication of the history book (b) proceeds not so much from
future (d) as because of a desire to shape perceptions of the past. (e) No error

(c) concern about the

Correct Answer: D
Answer Explanation:

Corrected Sentence:
Current opposition to the publication of the history book proceeds not so much from concern about the future as from a
desire to shape perceptions of the past.
The error in the sentence occurs at (D). The phrase as because of creates a lack of parallelism and should be replaced
with as from: oppositionproceeds not so much from [one thing]as from [another thing] So muchas is a
correlative construction, so its two parts must be parallel.
There is no error at (A). The adjective current appropriately modifies the subject of the sentence, the noun opposition.
In addition, the preposition to idiomatically follows opposition and appropriately introduces the prepositional phrase
to the publication.
There is no error at (B). The singular present tense verb proceeds agrees with the subject of the sentence, the singular
noun opposition, and appropriately conveys an action that is occurring in the present (current).
There is no error at (C). The noun concern appropriately serves as the object of the preposition from, and the
preposition about idiomatically follows concern and appropriately introduces the prepositional phrase about the
future.
There is an error in the sentence.

The College Board 2011. For the sole use of the person for whom this report has been provided. Not for duplication or distribution.

Page 24

PSAT/NMSQT

Questions and Answer Explanations 2010-2011

SATURDAY FORM

Writing Skills: Question 24


Difficulty Level: EASY (2 on a scale of 1-9)
Skill Category: Recognize Correctly Formed Sentences
Recognize correct sentence structure.
Question:

Select the one underlined part of the sentence that must be changed to make the sentence correct, or select "No error" to
indicate that the sentence contains no error.

The word seltzer, (a) now used (b) to refer to any carbonated water, once referred only to the effervescent mineral
water (c) obtained from natural springs (d) near the village of Nieder Selters, Germany. (e) No error

Correct Answer: E
Answer Explanation:

Corrected Sentence:
The word seltzer, now used to refer to any carbonated water, once referred only to the effervescent mineral water
obtained from natural springs near the village of Nieder Selters, Germany.
The correct choice is (E). There are no errors in the sentence.
There is no error at (A). The adverb now appropriately modifies the past participle used, which appropriately
modifies the subject of the sentence, the noun phrase The word seltzer.
There is no error at (B). The infinitive form of the verb, to refer, appropriately indicates what the word seltzer is now
used to do: to refer to any carbonated water.
There is no error at (C). The past participle obtained appropriately modifies the noun water; in addition, the
preposition from idiomatically follows obtained and appropriately introduces the prepositional phrase from natural
springs.
There is no error at (D). The preposition near idiomatically follows the noun phrase natural springs and appropriately
introduces the prepositional phrase near the village, which indicates where the springs were located.

The College Board 2011. For the sole use of the person for whom this report has been provided. Not for duplication or distribution.

Page 25

PSAT/NMSQT

Questions and Answer Explanations 2010-2011

SATURDAY FORM

Writing Skills: Question 25


Difficulty Level: MEDIUM (5 on a scale of 1-9)
Skill Category: Manage Word Choice and Grammatical Relationships Between Words
Understand relationships between and among words including subject-verb agreement, pronoun reference,
and verb form and tense.
Question:

Select the one underlined part of the sentence that must be changed to make the sentence correct, or select "No error" to
indicate that the sentence contains no error.

The Tale of Peter Rabbit , like all of Beatrix Potters 23 childrens books, (a) were printed in a small format (b) so
that children (c) could easily hold (d) it . (e) No error

Correct Answer: A
Answer Explanation:

Corrected Sentence:
The Tale of Peter Rabbit , like all of Beatrix Potters 23 childrens books, was printed in a small format so that children
could easily hold it.
The error in the sentence occurs at (A). The plural past tense verb were does not agree with the subject of the sentence,
the singular noun The Tale of Peter Rabbit. It should be replaced with the singular past tense verb was.
There is no error at (B). The conjunctive phrase so that appropriately joins the two independent clauses ( The Tale of
Peter Rabbit printed in a small format and children could easily hold it), indicating why the book was printed in a
small format.
There is no error at (C). The past tense verb phrase could...hold appropriately indicates what children were able to do as
a result of the book being printed in a small format. The adverb easily appropriately modifies hold.
There is no error at (D). The singular pronoun it appropriately refers to the subject of the sentence, the singular noun
The Tale of Peter Rabbit .
There is an error in the sentence.

The College Board 2011. For the sole use of the person for whom this report has been provided. Not for duplication or distribution.

Page 26

PSAT/NMSQT

Questions and Answer Explanations 2010-2011

SATURDAY FORM

Writing Skills: Question 26


Difficulty Level: MEDIUM (4 on a scale of 1-9)
Skill Category: Manage Word Choice and Grammatical Relationships Between Words
Understand relationships between and among words including subject-verb agreement, pronoun reference,
and verb form and tense.
Question:

Select the one underlined part of the sentence that must be changed to make the sentence correct, or select "No error" to
indicate that the sentence contains no error.

From 1851, (a) when (b) she joined the Underground Railroad, (c) and the beginning of the Civil War in 1861,
Harriet Tubman (d) helped nearly 300 people escape slavery. (e) No error

Correct Answer: C
Answer Explanation:

Corrected Sentence:
From 1851, when she joined the Underground Railroad, to the beginning of the Civil War in 1861, Harriet Tubman helped
nearly 300 people escape slavery.
The error in the sentence occurs at (C). The conjunction and is not idiomatic in this context. The modifying clause
From 1851... serves to indicate the time period during which Harriet Tubman helped nearly 300 people escape
slavery, but From 1851and1861 does not represent a time period. From 1851and...1861 should be replaced
with From 1851to1861.
There is no error at (A). The relative pronoun when appropriately refers to the date 1851.
There is no error at (B). The pronoun she appropriately refers to the subject of the sentence, Harriet Tubman, and the
past tense verb joined appropriately indicates an action that occurred in the past (1851).
There is no error at (D). The past tense verb helped appropriately indicates an action that took place in the past (From
1851[to]1861), and the adverb nearly appropriately modifies the adjective 300, which indicates the number of
people Harriet Tubman helped.
There is an error in the sentence.

The College Board 2011. For the sole use of the person for whom this report has been provided. Not for duplication or distribution.

Page 27

PSAT/NMSQT

Questions and Answer Explanations 2010-2011

SATURDAY FORM

Writing Skills: Question 27


Difficulty Level: MEDIUM (5 on a scale of 1-9)
Skill Category: Manage Grammatical Structures Used to Modify or Compare
Understand correct use of adjectives or adverbs, comparative structures (such as neither and nor), and
phrases used to modify or compare.
Question:

Select the one underlined part of the sentence that must be changed to make the sentence correct, or select "No error" to
indicate that the sentence contains no error.

(a) Because readers responded so (b) positive to her humorous newspaper columns about suburban life, Erma
Bombeck (c) wrote a number of books (d) on the subject. (e) No error

Correct Answer: B
Answer Explanation:

Corrected Sentence:
Because readers responded so positively to her humorous newspaper columns about suburban life, Erma Bombeck wrote a
number of books on the subject.
The error in the sentence occurs at (B). The underlined word, which indicates how readers responded to Bombecks
columns, modifies a verb, responded. Only an adverb can modify a verb, so the underlined word must be an adverb.
Positive is an adjective. It must be replaced with positively, an adverb.
There is no error at (A). The conjunction Because appropriately establishes the causal relationship between the two
clauses (readers respondedabout suburban life and Erma Bombeck wrote a number of books on the subject).
There is no error at (C). The past tense verb wrote is consistent with the previous past tense verb responded.
There is no error at (D). The preposition on idiomatically follows the noun books and appropriately introduces the
prepositional phrase on the subject.
There is an error in the sentence.

The College Board 2011. For the sole use of the person for whom this report has been provided. Not for duplication or distribution.

Page 28

PSAT/NMSQT

Questions and Answer Explanations 2010-2011

SATURDAY FORM

Writing Skills: Question 28


Difficulty Level: MEDIUM (6 on a scale of 1-9)
Skill Category: Manage Word Choice and Grammatical Relationships Between Words
Understand relationships between and among words including subject-verb agreement, pronoun reference,
and verb form and tense.
Question:

Select the one underlined part of the sentence that must be changed to make the sentence correct, or select "No error" to
indicate that the sentence contains no error.

It is (a) imperative that the government not censor this exhibit, (b) no matter how offensive the images
, but (d) continues to protect artists freedom of expression. (e) No error

(c) may be

Correct Answer: D
Answer Explanation:

Corrected Sentence:
It is imperative that the government not censor this exhibit, no matter how offensive the images may be, but continue to
protect artists freedom of expression.
The error in the sentence occurs at (D). The structure of the sentence (It is imperative that the governmentbut)
requires not the present tense singular verb form (censors, continues), but the infinitive form of the verb ([to]
censor or [to] continue), where the to part of the infinitive is implied. Also, the two verb forms must be parallel, and
continue is consistent with censor. Another way to look at the issue is this: saying It is imperative that the
government not censorbut continue is equivalent to saying The government must not censorbut continue"the
verb forms are the same in each case.
There is no error at (A). The adjective imperative, which means important or necessary, appropriately modifies the
lengthy noun clause that begins that the government In other words, that the governmentprotect artists freedom
of expression is imperative.
There is no error at (B). The phrase no matter how appropriately links the dependent clause (no matter howmay be)
to the main clause (It is imperative that the government not censor this exhibit) to indicate that censorship must not
occur, regardless of the potential offensiveness of the images (no matter how offensive the images may be).
There is no error at (C). The verb phrase may be appropriately indicates that regardless of the potential offensiveness
of the images (no matter how offensive the images may be), censorship must not occur.
There is an error in the sentence.

The College Board 2011. For the sole use of the person for whom this report has been provided. Not for duplication or distribution.

Page 29

PSAT/NMSQT

Questions and Answer Explanations 2010-2011

SATURDAY FORM

Writing Skills: Question 29


Difficulty Level: MEDIUM (5 on a scale of 1-9)
Skill Category: Manage Phrases and Clauses in a sentence
Use well-formed sentence structures (e.g., parallelism, connectives, and relative clauses) to indicate
relationships between and among sentence elements.
Question:

Select the one underlined part of the sentence that must be changed to make the sentence correct, or select "No error" to
indicate that the sentence contains no error.

The theory of continental drift, which (a) originated in 1912 and holds that Earths continents move continually in
relation (b) to one another , (c) it became more plausible (d) as new geological evidence emerged in the 1950s.
(e) No error

Correct Answer: C
Answer Explanation:

Corrected Sentence:
The theory of continental drift, which originated in 1912 and holds that Earths continents move continually in relation to
one another, became more plausible as new geological evidence emerged in the 1950s.
The error in the sentence occurs at (C). The pronoun it is redundant in this context, unnecessarily restating the subject
The theory of continental drift, and should be deleted.
There is no error at (A). The past tense verb originated appropriately describes an action that occurred in the past, and
the preposition in appropriately introduces the prepositional phrase in 1912, which indicates when the theory
originated.
There is no error at (B). The preposition to idiomatically follows the noun relation and appropriately introduces the
prepositional phrase to one another. The reciprocal pronoun one another appropriately describes the interactions of
more than two continents (each other would be required if only two continents were being discussed).
There is no error at (D). The conjunction as appropriately introduces a subordinate clause (as new geological evidence
emerged in the 1950s) that explains why The theory of continental driftbecame more plausible.
There is an error in the sentence.

The College Board 2011. For the sole use of the person for whom this report has been provided. Not for duplication or distribution.

Page 30

PSAT/NMSQT

Questions and Answer Explanations 2010-2011

SATURDAY FORM

Writing Skills: Question 30


Difficulty Level: MEDIUM (5 on a scale of 1-9)
Skill Category: Recognize Correctly Formed Sentences
Recognize correct sentence structure.
Question:

Select the one underlined part of the sentence that must be changed to make the sentence correct, or select "No error" to
indicate that the sentence contains no error.

In her new play, Eurydice , playwright Sarah Ruhl has concentrated (a) not on the passionate (b) journey of
Orpheus (c) to retrieve his bride (d) but on Eurydices descent into the underworld. (e) No error

Correct Answer: E
Answer Explanation:

Corrected Sentence:
In her new play, Eurydice , playwright Sarah Ruhl has concentrated not on the passionate journey of Orpheus to retrieve
his bride but on Eurydices descent into the underworld.
Choice (E) is correct. There are no errors in the sentence.
There is no error at (A). The conjunction not appropriately introduces a not [this]but [that] construction. In
addition, the preposition on idiomatically follows the verb concentrated and appropriately precedes the noun phrase
the passionate journey of Orpheus to retrieve his bride.
There is no error at (B). The noun journey appropriately indicates what the playwright has not concentrated on, and the
preposition of appropriately introduces the prepositional phrase of Orpheus.
There is no error at (C). The infinitive form of the verb, to retrieve, appropriately indicates why Orpheus took the
journey: to retrieve his bride.
There is no error at (D). The conjunction but appropriately completes the not [this]but [that] construction. In
addition, the preposition on idiomatically follows the verb concentrated and appropriately precedes the noun phrase
Eurydices descent into the underworld.

The College Board 2011. For the sole use of the person for whom this report has been provided. Not for duplication or distribution.

Page 31

PSAT/NMSQT

Questions and Answer Explanations 2010-2011

SATURDAY FORM

Writing Skills: Question 31


Difficulty Level: MEDIUM (6 on a scale of 1-9)
Skill Category: Manage Word Choice and Grammatical Relationships Between Words
Understand relationships between and among words including subject-verb agreement, pronoun reference,
and verb form and tense.
Question:

Select the one underlined part of the sentence that must be changed to make the sentence correct, or select "No error" to
indicate that the sentence contains no error.

Vigorous student (a) debate with the problem of (b) steeply increased student activity fees
people to take a critical look at the activities (d) those fees support . (e) No error

(c) has caused many

Correct Answer: A
Answer Explanation:

Corrected Sentence:
Vigorous student debate over the problem of steeply increased student activity fees has caused many people to take a
critical look at the activities those fees support.
The error in the sentence occurs at (A). The preposition with is not idiomatic following the noun debate and should be
replaced with an idiomatic preposition such as over or about.
There is no error at (B). The adverb steeply appropriately modifies the past participle increased, which serves as an
adjective modifying the noun phrase student activity fees.
There is no error at (C). The singular present perfect verb phrase has caused agrees with the subject of the sentence, the
singular noun debate, and appropriately indicates an action that is recent or ongoing.
There is no error at (D). The noun phrase those fees appropriately refers to the student activity fees mentioned earlier
in the sentence, and the plural verb support agrees with the plural noun fees. The phrase those fees is preferable to
the pronoun they in this context because they would be ambiguous (it could refer to either fees or people).
There is an error in the sentence.

The College Board 2011. For the sole use of the person for whom this report has been provided. Not for duplication or distribution.

Page 32

PSAT/NMSQT

Questions and Answer Explanations 2010-2011

SATURDAY FORM

Writing Skills: Question 32


Difficulty Level: HARD (8 on a scale of 1-9)
Skill Category: Manage Word Choice and Grammatical Relationships Between Words
Understand relationships between and among words including subject-verb agreement, pronoun reference,
and verb form and tense.
Question:

Select the one underlined part of the sentence that must be changed to make the sentence correct, or select "No error" to
indicate that the sentence contains no error.

(a) Running for the office of mayor (b) were four candidates, three of whom
member of the city council. (e) No error

(c) had all recently served as

(d) a

Correct Answer: D
Answer Explanation:

Corrected Sentence:
Running for the office of mayor were four candidates, three of whom had all recently served as members of the city
council.
The error in the sentence occurs at (D). The sentence refers to the three candidates in the plural form rather than
considering each candidate separately: they (three of [the candidates]) had all recently served on the city council.
Thus, the singular noun phrase a member should be replaced with the plural noun members to refer to the candidates.
There is no error at (A). The main clause of the sentence (Running for the office of mayor were four candidates)
appropriately inverts the subject and verb of the clause Four candidates were running for the office of mayor in order to
place the modifying clause three of whom next to the noun it modifies, candidates.
There is no error at (B). The main clause of the sentence (Running for the office of mayor were four candidates)
appropriately inverts the subject and verb of the clause Four candidates were running for the office of mayor in order to
place the modifying clause three of whom next to the noun it modifies, candidates.
There is no error at (C). The past perfect verb phrase hadserved appropriately indicates an action that was completed
before another action in the past (Running for the office of mayor were four candidates).
There is an error in the sentence.

The College Board 2011. For the sole use of the person for whom this report has been provided. Not for duplication or distribution.

Page 33

PSAT/NMSQT

Questions and Answer Explanations 2010-2011

SATURDAY FORM

Writing Skills: Question 33


Difficulty Level: HARD (8 on a scale of 1-9)
Skill Category: Manage Word Choice and Grammatical Relationships Between Words
Understand relationships between and among words including subject-verb agreement, pronoun reference,
and verb form and tense.
Question:

Select the one underlined part of the sentence that must be changed to make the sentence correct, or select "No error" to
indicate that the sentence contains no error.

Although Africas forest elephants and savanna elephants (a) were once (b) thought to belong to the same species, a
recent study of the animals skeletons and tusks (c) suggest (d) otherwise . (e) No error

Correct Answer: C
Answer Explanation:

Corrected Sentence:
Although Africas forest elephants and savanna elephants were once thought to belong to the same species, a recent study
of the animals skeletons and tusks suggests otherwise.
The error in the sentence occurs at (C). The subject of the sentence, study, is a singular noun, but the main verb of the
sentence, suggest, is plural. The plural verb suggest should be replaced with the singular verb suggests.
There is no error at (A). The plural past tense verb phrase werethought agrees with the plural noun elephants and
appropriately indicates an action that occurred in the past (before the recent study). The adverb once, which in this
context means at some indefinite time in the past, appropriately indicates when elephants werethought to belong to
the same species.
There is no error at (B). The past tense verb phrase werethought appropriately indicates an action that occurred in the
past (before the recent study). The infinitive form of the verb, to belong, idiomatically indicates what was once
thought about the elephants: they were once thought to belong .
There is no error at (D). The pronoun otherwise, which in this context means something different or to the contrary,
appropriately serves as the object of the verb suggest[s].
There is an error in the sentence.

The College Board 2011. For the sole use of the person for whom this report has been provided. Not for duplication or distribution.

Page 34

PSAT/NMSQT

Questions and Answer Explanations 2010-2011

SATURDAY FORM

Writing Skills: Question 34


Difficulty Level: HARD (8 on a scale of 1-9)
Skill Category: Manage Phrases and Clauses in a sentence
Use well-formed sentence structures (e.g., parallelism, connectives, and relative clauses) to indicate
relationships between and among sentence elements.
Question:

Select the one underlined part of the sentence that must be changed to make the sentence correct, or select "No error" to
indicate that the sentence contains no error.

The first woman in the United States (a) to be granted a patent was Mary Dixon Kies,
weaving straw with other fibers proved (d) useful to hatmakers. (e) No error

(b) her new method for

(c)

Correct Answer: B
Answer Explanation:

Corrected Sentence:
The first woman in the United States to be granted a patent was Mary Dixon Kies, whose new method for weaving straw
with other fibers proved useful to hatmakers.
The error in the sentence occurs at (B). The use of the pronoun her after Mary Dixon Kies results in a comma splice;
two independent clauses (The first womanMary Dixon Kies and her new methodproved useful to hatmakers) are
improperly joined by only a comma. The second clause of the sentence, which modifies Mary Dixon Kies, should begin
with the relative pronoun whose. The error could also be fixed, although less effectively, by adding a conjunction such
as because or and before her.
There is no error at (A). The infinitive form of the verb, to be, when joined with the past participle granted,
appropriately indicates what Mary Dixon Kies was the first U.S. woman to do: she was the first to be granted a patent
by the government.
There is no error at (C). The gerund weaving, which is the object of the preposition for, and straw, which is the
object of the gerund weaving, join to create a prepositional phrase (for weaving straw) that appropriately modifies the
noun method and indicates what Kies patented: a new method for weaving straw
There is no error at (D). In this context, the past tense verb proved means showed a particular quality. The adjective
useful indicates the particular quality demonstrated by Kiess method and thus appropriately follows proved. The
prepositional phrase to hatmakers indicates to whom the method showed the particular quality; hatmakers thus
functions appropriately as the object of the preposition to.
There is an error in the sentence.

The College Board 2011. For the sole use of the person for whom this report has been provided. Not for duplication or distribution.

Page 35

PSAT/NMSQT

Questions and Answer Explanations 2010-2011

SATURDAY FORM

Writing Skills: Question 35


Difficulty Level: MEDIUM (4 on a scale of 1-9)
Skill Category: Manage Order and Relationships of Sentences and Paragraphs
Identify how to order the elements of a sentence or paragraph to improve clarity, meaning, and the progression
of ideas.
Passage 1
(1) At the beginning of the twentieth century, textile workers in Lowell, Massachusetts,
worked and lived under deplorable conditions. (2) The average worker earned between $6 and
$8 for a 56-hour workweek and lived in an overcrowded slum. (3) Industrial accidents
occurred, and workers suffered injuries. (4) This might have resulted in the fact that more than
33 percent of all mill workers died before they reached the age of 26. (5) As if conditions were
not bad enough, unscrupulous employers took advantage of workers.
(6) In 1912 the Massachusetts legislature reduced the workweek to 54 hours per week but did
not call for a commensurate increase in hourly wages. (7) Without warning workers, the
employers simply cut the workers pay after the Massachusetts law went into effect. (8) To the
surprise of union organizers, most of whom had believed that the mill workers were too poor
and uneducated to be unionized, workers instantly went on strike. (9) Members of the
Industrial Workers of the World labor union (IWW) arrived to help the mill workers conduct
an effective strike.
(10) Strikers destroyed machines in the factories, and there were numerous clashes between
strikers and the police. (11) In one incident, striking workers had arranged to have their
children cared for by outside sympathizers and transported by train to safety out of town. (12)
Authorities had forbidden the children to be sent away, and police were ordered to attack
families arriving at the train station. (13) Photographs of the attacks were a good way to turn
public opinion against the mill owners and create pressure on the two sides to come to an
agreement.

Question:

In context, which of the following is the best way to revise and combine sentences 3 and 4 (reproduced below) ?

Industrial accidents occurred, and workers suffered injuries. This might have resulted in the fact that more than 33 percent
of all mill workers died before they reached the age of 26.

(A) Industrial accidents occurred and injuries were suffered by workers; however, more than 33 percent of all
mill workers died before they reached the age of 26.
(B) Industrial accidents occurred, and injuries were suffered by workers; a result of the fact that more than 33
percent of all mill workers died before reaching the age of 26.
(C) More than 33 percent of all mill workers died before they reached the age of 26, possibly as the result of
injuries suffered in industrial accidents.
(D) More than 33 percent of mill workers died before they reached the age of 26, but there were industrial
accidents and workers suffered injuries.
(E) With industrial accidents occurring and workers suffering injuries, this could have resulted from the fact
that more than 33 percent of all mill workers died before they reached the age of 26.
Answer Explanation:
The College Board 2011. For the sole use of the person for whom this report has been provided. Not for duplication or distribution.

Page 36

PSAT/NMSQT

Questions and Answer Explanations 2010-2011

SATURDAY FORM

Choice (C) is correct. This sentence is the only option to link the deaths (More than 33 percent of all mill workers died
before they reached the age of 26) to their likely cause (possibly as a result of injuries suffered in industrial accidents),
clearly and logically.
Choice (A) is unsatisfactory. The word however should be used to indicate a contrast between two clauses, but the two
clauses in this sentence (Industrial accidents occurred and injuries were suffered by workers and more than 33 percent
of all mill workers died before they reached the age of 26) do not form a contrast. Rather, the two clauses are closely
related: the deaths seem to be a result of the accidents and injuries.
Choice (B) is unsatisfactory. The sentence uses a semicolon to join an independent clause (Industrial accidents occurred,
and injuries were suffered by workers) with a dependent clause (a result of the fact that more than 33 percent of all mill
workers died before they reached the age of 26). Semicolons should be used to connect two independent clauses, not an
independent clause and dependent clause. In addition, what the sentence seems to be suggesting does not make sense. In
other words, it does not make sense to say that industrial accidents and injuries were a result of the fact that more
than 33 percent of all mill workers died
Choice (D) is unsatisfactory. The word but should be used to indicate a contrast between two clauses, but the two
clauses in this sentence (More than 33 percent of mill workers died before they reached the age of 26) and (there were
industrial accidents and workers suffered injuries) do not form a contrast. Rather, the two clauses are closely related: the
deaths seem to be a result of the accidents and injuries.
Choice (E) is unsatisfactory. The sentence does not make sense. It seems to indicate that the injuries were a result of the
deaths. There is also nothing in the sentence to which this could logically refer, since a pronoun cannot refer to a
prepositional phrase like With industrial accidents occurring and workers suffering injuries.

The College Board 2011. For the sole use of the person for whom this report has been provided. Not for duplication or distribution.

Page 37

PSAT/NMSQT

Questions and Answer Explanations 2010-2011

SATURDAY FORM

Writing Skills: Question 36


Difficulty Level: MEDIUM (6 on a scale of 1-9)
Skill Category: Manage Order and Relationships of Sentences and Paragraphs
Identify how to order the elements of a sentence or paragraph to improve clarity, meaning, and the progression
of ideas.
Passage 1
(1) At the beginning of the twentieth century, textile workers in Lowell, Massachusetts,
worked and lived under deplorable conditions. (2) The average worker earned between $6 and
$8 for a 56-hour workweek and lived in an overcrowded slum. (3) Industrial accidents
occurred, and workers suffered injuries. (4) This might have resulted in the fact that more than
33 percent of all mill workers died before they reached the age of 26. (5) As if conditions were
not bad enough, unscrupulous employers took advantage of workers.
(6) In 1912 the Massachusetts legislature reduced the workweek to 54 hours per week but did
not call for a commensurate increase in hourly wages. (7) Without warning workers, the
employers simply cut the workers pay after the Massachusetts law went into effect. (8) To the
surprise of union organizers, most of whom had believed that the mill workers were too poor
and uneducated to be unionized, workers instantly went on strike. (9) Members of the
Industrial Workers of the World labor union (IWW) arrived to help the mill workers conduct
an effective strike.
(10) Strikers destroyed machines in the factories, and there were numerous clashes between
strikers and the police. (11) In one incident, striking workers had arranged to have their
children cared for by outside sympathizers and transported by train to safety out of town. (12)
Authorities had forbidden the children to be sent away, and police were ordered to attack
families arriving at the train station. (13) Photographs of the attacks were a good way to turn
public opinion against the mill owners and create pressure on the two sides to come to an
agreement.

Question:

Which of the following sentences, if placed after sentence 5, would most effectively conclude the first paragraph?

(A) Changes occurred throughout the past century in the relationships between employers and employees.
(B) Originally, some mill owners had wanted to create a positive living and working environment for the
workers.
(C) Lowell had been a flourishing industrial city from as early as the 1830s.
(D) The mill owners charged money for drinking water and docked workers an hours pay for arriving just a
few minutes late.
(E) Many of the mill workers were unskilled, but some were skilled laborers.
Answer Explanation:

Choice (D) is correct. Sentence 5 extends the discussion of deplorable conditions in sentences 1-4 (As if conditions
were not bad enough) by mentioning an additional problem for workers: unscrupulous employers took advantage of
workers. Only choice (D) gives information about this additional problem: charg[ing] money for drinking water and
dock[ing] workers an hours pay for arriving just a few minutes late are both good examples of workers being taken
advantage of by their bosses.
The College Board 2011. For the sole use of the person for whom this report has been provided. Not for duplication or distribution.

Page 38

PSAT/NMSQT

Questions and Answer Explanations 2010-2011

SATURDAY FORM

Choice (A) is unsatisfactory. Sentence 5 extends the discussion of deplorable conditions in sentences 1-4 (As if
conditions were not bad enough) by mentioning an additional problem for workers: unscrupulous employers took
advantage of workers. Choice (A) does not discuss deplorable conditions or unscrupulous employers, only
[c]hangesthroughout the past century in the relationships between employers and employees.
Choice (B) is unsatisfactory. Sentence 5 extends the discussion of deplorable conditions in sentences 1-4 (As if
conditions were not bad enough) by mentioning an additional problem for workers: unscrupulous employers took
advantage of workers. Choice (B) does not discuss deplorable conditions or unscrupulous employers, only that some
mill owners had wanted to create a positive living and working environment for the workers.
Choice (C) is unsatisfactory. Sentence 5 extends the discussion of deplorable conditions in sentences 1-4 (As if
conditions were not bad enough) by mentioning an additional problem for workers: unscrupulous employers took
advantage of workers. Choice (C) does not discuss deplorable conditions or unscrupulous employers, only that Lowell
had been a flourishing industrial city from as early as the 1830s.
Choice (E) is unsatisfactory. Sentence 5 extends the discussion of deplorable conditions in sentences 1-4 (As if
conditions were not bad enough) by mentioning an additional problem for workers: unscrupulous employers took
advantage of workers. Choice (E) does not discuss deplorable conditions or unscrupulous employers, only that [m]any
of the mill workers were unskilled, but some were skilled laborers.

The College Board 2011. For the sole use of the person for whom this report has been provided. Not for duplication or distribution.

Page 39

PSAT/NMSQT

Questions and Answer Explanations 2010-2011

SATURDAY FORM

Writing Skills: Question 37


Difficulty Level: MEDIUM (6 on a scale of 1-9)
Skill Category: Manage Order and Relationships of Sentences and Paragraphs
Identify how to order the elements of a sentence or paragraph to improve clarity, meaning, and the progression
of ideas.
Passage 1
(1) At the beginning of the twentieth century, textile workers in Lowell, Massachusetts,
worked and lived under deplorable conditions. (2) The average worker earned between $6 and
$8 for a 56-hour workweek and lived in an overcrowded slum. (3) Industrial accidents
occurred, and workers suffered injuries. (4) This might have resulted in the fact that more than
33 percent of all mill workers died before they reached the age of 26. (5) As if conditions were
not bad enough, unscrupulous employers took advantage of workers.
(6) In 1912 the Massachusetts legislature reduced the workweek to 54 hours per week but did
not call for a commensurate increase in hourly wages. (7) Without warning workers, the
employers simply cut the workers pay after the Massachusetts law went into effect. (8) To the
surprise of union organizers, most of whom had believed that the mill workers were too poor
and uneducated to be unionized, workers instantly went on strike. (9) Members of the
Industrial Workers of the World labor union (IWW) arrived to help the mill workers conduct
an effective strike.
(10) Strikers destroyed machines in the factories, and there were numerous clashes between
strikers and the police. (11) In one incident, striking workers had arranged to have their
children cared for by outside sympathizers and transported by train to safety out of town. (12)
Authorities had forbidden the children to be sent away, and police were ordered to attack
families arriving at the train station. (13) Photographs of the attacks were a good way to turn
public opinion against the mill owners and create pressure on the two sides to come to an
agreement.

Question:

In context, which of the following sentences would be best to place immediately after sentence 9 ?

(A) The union provided meals for workers families and helped organize picket lines.
(B) Union organizers were merely expressing popular beliefs about the economic and educational status of the
mill workers.
(C) The mill owners, however, had the law behind them.
(D) The IWW has been written about in many history books.
(E) This made the lives of the workers harder, but they felt strongly about their cause.
Answer Explanation:

Choice (A) is correct. Sentences 8 and 9 explain that after the Lowell workers, who were believed to be too poor and
uneducated to be unionized, began their strike, [m]embers of thelabor union (IWW) arrived to help the mill workers
conduct an effective strike. Choice (A) is the only option to give specific information about how the union helped: The
union provided meals for workers families and helped organize picket lines.
Choice (B) is unsatisfactory. Sentence 8 explains that even though the Lowell workers were believed to be too poor and
The College Board 2011. For the sole use of the person for whom this report has been provided. Not for duplication or distribution.

Page 40

PSAT/NMSQT

Questions and Answer Explanations 2010-2011

SATURDAY FORM

uneducated to be unionized, they went on strike after their wages were cut. Sentence 9 follows logically by stating that
Members of thelabor union (IWW) arrived to help the mill workers conduct an effective strike. Since choice (B)
discusses the union organizers beliefs, not the help they provided, it would best fit immediately after sentence 8, not
sentence 9. Choice (A) is the only option to give specific information about how the union helped: The union provided
meals for workers families and helped organize picket lines.
Choice (C) is unsatisfactory. Sentences 8 and 9 explain that after the workers, who were believed to be too poor and
uneducated to be unionized, began their strike, [m]embers of thelabor union (IWW) arrived to help the mill workers
conduct an effective strike. Choice (A) is the only option to give specific information about how the union helped: The
union provided meals for workers families and helped organize picket lines. It would be out of place to mention mill
owners at this point in the discussion.
Choice (D) is unsatisfactory. Sentences 8 and 9 explain that after the workers, who were believed to be too poor and
uneducated to be unionized, began their strike, [m]embers of thelabor union (IWW) arrived to help the mill workers
conduct an effective strike. Choice (A) is the only option to give specific information about how the union helped: The
union provided meals for workers families and helped organize picket lines. It would be out of place to mention the
history of the IWW at this point in the discussion.
Choice (E) is unsatisfactory. Sentences 8 and 9 explain that after the workers, who were believed to be too poor and
uneducated to be unionized, began their strike, [m]embers of thelabor union (IWW) arrived to help the mill workers
conduct an effective strike. Choice (A) is the only option to give specific information about how the union helped: The
union provided meals for workers families and helped organize picket lines. It would be illogical to say that the lives of
workers who were already on strike would be made harder with the help of experienced union organizers.

The College Board 2011. For the sole use of the person for whom this report has been provided. Not for duplication or distribution.

Page 41

PSAT/NMSQT

Questions and Answer Explanations 2010-2011

SATURDAY FORM

Writing Skills: Question 38


Difficulty Level: EASY (3 on a scale of 1-9)
Skill Category: Manage Order and Relationships of Sentences and Paragraphs
Identify how to order the elements of a sentence or paragraph to improve clarity, meaning, and the progression
of ideas.
Passage 1
(1) At the beginning of the twentieth century, textile workers in Lowell, Massachusetts,
worked and lived under deplorable conditions. (2) The average worker earned between $6 and
$8 for a 56-hour workweek and lived in an overcrowded slum. (3) Industrial accidents
occurred, and workers suffered injuries. (4) This might have resulted in the fact that more than
33 percent of all mill workers died before they reached the age of 26. (5) As if conditions were
not bad enough, unscrupulous employers took advantage of workers.
(6) In 1912 the Massachusetts legislature reduced the workweek to 54 hours per week but did
not call for a commensurate increase in hourly wages. (7) Without warning workers, the
employers simply cut the workers pay after the Massachusetts law went into effect. (8) To the
surprise of union organizers, most of whom had believed that the mill workers were too poor
and uneducated to be unionized, workers instantly went on strike. (9) Members of the
Industrial Workers of the World labor union (IWW) arrived to help the mill workers conduct
an effective strike.
(10) Strikers destroyed machines in the factories, and there were numerous clashes between
strikers and the police. (11) In one incident, striking workers had arranged to have their
children cared for by outside sympathizers and transported by train to safety out of town. (12)
Authorities had forbidden the children to be sent away, and police were ordered to attack
families arriving at the train station. (13) Photographs of the attacks were a good way to turn
public opinion against the mill owners and create pressure on the two sides to come to an
agreement.

Question:

In context, which of the following sentences would best be placed at the beginning of paragraph 3 (sentences 1013) ?

(A) Violence is never a good solution to problems in the workplace.


(B) Public opinion was on the side of the striking workers from the beginning.
(C) Whatever the intentions of the union organizers might have been, violence broke out.
(D) Besides, it was inevitable that eventually, clashes would take place.
(E) So what happened next would probably not surprise you.
Answer Explanation:

Choice (C) is correct. The second paragraph ends with arrival of the union organizers to help the mill workers conduct an
effective strike. The third paragraph focuses on the numerous clashes between the strikers and the police. Choice (C) is
the only option that incorporates both ideas and makes a clear, complete transition: Whatever the intentions of the union
organizers may have been, violence broke out.
Choice (A) is unsatisfactory. The second paragraph ends with the arrival of the union organizers to help the mill workers
conduct an effective strike. The third paragraph focuses on the numerous clashes between the strikers and the police.
The College Board 2011. For the sole use of the person for whom this report has been provided. Not for duplication or distribution.

Page 42

PSAT/NMSQT

Questions and Answer Explanations 2010-2011

SATURDAY FORM

Choice (C) is the only option that incorporates both ideas and makes a clear, complete transition: Whatever the intentions
of the union organizers may have been, violence broke out. Choice (A) mentions violence, but leaves out the union
organizers. In addition, it is not clear if the outcome of the strike supports the claim that violence is never a good
solution; by the end of the paragraph, we learn that the violence helped the workers come to an agreement.
Choice (B) is unsatisfactory. The second paragraph ends with the arrival of the union organizers to help the mill workers
conduct an effective strike. The third paragraph focuses on the numerous clashes between the strikers and the police.
Choice (C) is the only option that incorporates both ideas and makes a clear, complete transition: Whatever the intentions
of the union organizers may have been, violence broke out. In addition, choice (B)s statement that Public opinion was
on the side of the striking workers from the beginning seems to be contradicted later in the third paragraph, in sentence
13: Photographs of the attacks were a good way to turn public opinion against the mill owners. If public opinion were
already on the workers side, it would not need to be turned.
Choice (D) is unsatisfactory. The second paragraph ends with the arrival of the union organizers to help the mill workers
conduct an effective strike. The third paragraph focuses on the numerous clashes between the strikers and the police.
Choice (C) is the only option that incorporates both ideas and makes a clear, complete transition: Whatever the intentions
of the union organizers may have been, violence broke out. Choice (D) mentions clashes, but leaves out the union
organizers. In addition, nothing in the passage supports the idea that the clashes were inevitable.
Choice (E) is unsatisfactory. The second paragraph ends with arrival of the union organizers to help the mill workers
conduct an effective strike. The third paragraph focuses on the numerous clashes between the strikers and the police.
Choice (C) is the only option that incorporates both ideas and makes a clear, complete transition: Whatever the intentions
of the union organizers may have been, violence broke out. In addition, choice (E) uses the second-person pronoun
you, which is not consistent with the passage, and makes an assumption about readers opinions (what happened next
would probably not surprise you) that seems unwarranted.

The College Board 2011. For the sole use of the person for whom this report has been provided. Not for duplication or distribution.

Page 43

PSAT/NMSQT

Questions and Answer Explanations 2010-2011

SATURDAY FORM

Writing Skills: Question 39


Difficulty Level: MEDIUM (4 on a scale of 1-9)
Skill Category: Manage Order and Relationships of Sentences and Paragraphs
Identify how to order the elements of a sentence or paragraph to improve clarity, meaning, and the progression
of ideas.
Passage 1
(1) At the beginning of the twentieth century, textile workers in Lowell, Massachusetts,
worked and lived under deplorable conditions. (2) The average worker earned between $6 and
$8 for a 56-hour workweek and lived in an overcrowded slum. (3) Industrial accidents
occurred, and workers suffered injuries. (4) This might have resulted in the fact that more than
33 percent of all mill workers died before they reached the age of 26. (5) As if conditions were
not bad enough, unscrupulous employers took advantage of workers.
(6) In 1912 the Massachusetts legislature reduced the workweek to 54 hours per week but did
not call for a commensurate increase in hourly wages. (7) Without warning workers, the
employers simply cut the workers pay after the Massachusetts law went into effect. (8) To the
surprise of union organizers, most of whom had believed that the mill workers were too poor
and uneducated to be unionized, workers instantly went on strike. (9) Members of the
Industrial Workers of the World labor union (IWW) arrived to help the mill workers conduct
an effective strike.
(10) Strikers destroyed machines in the factories, and there were numerous clashes between
strikers and the police. (11) In one incident, striking workers had arranged to have their
children cared for by outside sympathizers and transported by train to safety out of town. (12)
Authorities had forbidden the children to be sent away, and police were ordered to attack
families arriving at the train station. (13) Photographs of the attacks were a good way to turn
public opinion against the mill owners and create pressure on the two sides to come to an
agreement.

Question:

In context, which of the following is the best version of the underlined portion of sentence 13 (reproduced below) ?

Photographs of the attacks were a good way to turn public opinion against the mill owners and create pressure on the two
sides to come to an agreement.

(A) were good for turning


(B) made them turn
(C) helped turn
(D) are a way to turn
(E) turned
Answer Explanation:

Choice (C) is correct. Inserting the past tense verb phrase helped turn after Photographs of the attacks results in a
logical and grammatically correct sentence. Logically, it makes more sense to say that the photographs helped turn
than to say that the photographs were solely responsible for the changes, and grammatically, the verbs turn and create
The College Board 2011. For the sole use of the person for whom this report has been provided. Not for duplication or distribution.

Page 44

PSAT/NMSQT

Questions and Answer Explanations 2010-2011

SATURDAY FORM

are parallel after helped (helped turnand [helped] create).


Choice (A) is unsatisfactory. Inserting were good for turning after Photographs of the attacks results in an illogical
sentence. It is awkward to say that photographs were good for turningand create pressure; the sentence shifts into the
present tense (photographscreate), which makes no sense given the historical context of the passage (1912).
Choice (B) is unsatisfactory. Inserting made them turn after Photographs of the attacks results in an ungrammatical
sentence. There is nothing in the sentence to which the pronoun them can logically refer.
Choice (D) is unsatisfactory. Inserting are a way to turn after Photographs of the attacks results in an ungrammatical
sentence. The subject of the sentence, the plural noun photographs, cannot logically be described as a way. In
addition, the action of the sentence occurred in 1912, which requires a past tense verb, not the present tense verb are.
Choice (E) is unsatisfactory. Inserting turned after Photographs of the attacks results in an illogical sentence. It is
awkward to say that photographs turnedand create pressure; the sentence shifts into the present tense
(photographscreate), which makes no sense given the historical context of the passage (1912).

The College Board 2011. For the sole use of the person for whom this report has been provided. Not for duplication or distribution.

Page 45

You might also like